Đến nội dung

PDF nội dung

Có 197 mục bởi PDF (Tìm giới hạn từ 09-05-2020)



Sắp theo                Sắp xếp  

#723434 BĐT AM-GM

Đã gửi bởi PDF on 01-07-2019 - 21:56 trong Bất đẳng thức và cực trị

Mình cũng có bài gần tương tự với bài VMO 1996:

Cho $x,y,z\geq 0$ thỏa mãn: $xy+yz+zx+6xyz=9$ . Chứng minh rằng:

$2(x+y+z)\geq xy+yz+zx+3$




#722344 Topic về bất đẳng thức

Đã gửi bởi PDF on 18-05-2019 - 21:15 trong Bất đẳng thức - Cực trị

Giúp mình bài này với:

Cho a;b;c dương thỏa mãn a+b+c=ab+bc+ca.

CMR: $\sum \frac{a+b}{a^{2}+b^{2}}\leq 3$




#728761 [TOPIC] ÔN THI SỐ HỌC VÀO THPT CHUYÊN NĂM 2020-2021

Đã gửi bởi PDF on 10-07-2021 - 21:04 trong Số học

Bài 150: Tìm các số nguyên dương $a$ để với mọi số nguyên tố lẻ $p$ luôn tồn tại số nguyên dương $n$ thỏa mãn đồng thời $a^n-n^2$ và $a^{n+1}-(n+1)^{2}$ đều chia hết cho $p$

Gợi ý




#728285 [TOPIC] ÔN TẬP HÌNH HỌC THI VÀO THPT CHUYÊN 2020-2021

Đã gửi bởi PDF on 20-06-2021 - 15:26 trong Hình học

$\boxed{48}$ Cho tam giác $ABC$ nội tiếp đường tròn $(O)$. Trên $OA$ lấy $P$ tùy ý. Gọi $E,F$ tương ứng là hình chiếu vuông góc của $P$ lên$ AC, AB$. Xét điểm $Q$ di động trên đoạn thẳng $EF$. Đường thẳng vuông góc với $AQ$ tại $Q$ cắt $PE,PF$ lần lượt tại $M,N$. Gọi $K$ là tâm đường tròn ngoại tiếp tam giác $AMN$ và gọi $D$ là hình chiếu vuông góc của $K$ lên $BC$. Chứng minh đường thẳng qua $D$ và song song với $AQ$ luôn đi qua $1$ điểm cố định.

Source: Underfined

Gợi ý: Sử dụng phương pháp suy biến (đưa Q đến vị trí đặc biệt).




#728287 [TOPIC] ÔN TẬP HÌNH HỌC THI VÀO THPT CHUYÊN 2020-2021

Đã gửi bởi PDF on 20-06-2021 - 15:46 trong Hình học

Sắp tới có thể mình sẽ lập thêm 1 Topic hình học phẳng 10 nữa nên mong các bạn ủng hộ  :D 

$\boxed{47}$ Cho tam giác $ABC$ vuông tại $A$ với $D$ là hình chiếu vuông góc của $A$ trên $BC$. Gọi $E, F, J$ lần lượt là tâm các đường tròn bàng tiếp $wrt$ đỉnh $A$ của các tam giác $ABD, ACD, ABC$ tương ứng. Gọi $P$ là hình chiếu vuông góc của $J$ lên $BC$ và $K$ là trung điểm của $EF$ .

a) Chứng minh rằng $KD=KP$

b) Chứng minh rằng tâm đường tròn $Euler$ của tam giác $AEF$ nằm trên đường thẳng $AD$ .

Source: Underfined

Lời giải khác:

a) Bằng biến đổi góc, ta chứng minh được $AEJF$ là hình bình hành (để ý hai tứ giác nội tiếp $BDFJ,CDEJ$).

Suy ra $K$ là trung điểm $AJ$. Do đó $KD=KP$ theo bổ đề hình thang vuông dạng xoắn.

 

b) Để ý rằng tứ giác $PDEF$ nội tiếp, mà $\overleftrightarrow{AD}$ là đường phân giác trong góc $EDF$ nên $DP$ là đường phân giác ngoài, suy ra $PE=PF$. Lại có $\angle EPF=90^{\circ}=2\angle EAF$ nên $P$ là tâm $(AEF)$.

Gọi $G$ là trọng tâm tam giác $AEF$. Khi đó $G$ thuộc $AK$. $PG$ cắt $AD$ tại $O_{9}$.

Khi đó $$\frac{GO_{9}}{GP}=\frac{GA}{GJ}=\frac{1}{2},$$

suy ra $O_{9}$ là tâm đường tròn Euler của tam giác $AEF$. $\square$




#726343 [TOPIC] ÔN TẬP HÌNH HỌC THI VÀO THPT CHUYÊN 2020-2021

Đã gửi bởi PDF on 02-05-2021 - 10:11 trong Hình học

Để ý $U,U',V,V'$, $U,U',W,W'$ đồng viên, sau đấy biến đổi góc.

$\boxed{40}$Cho tam giác ABC trực tâm H, có điểm P bất kì. U là giao điểm của đường thẳng vuông góc với AP qua H với BC. Cách xác định điểm V, W tương tự điểm U. Chứng minh rằng U,W,V thẳng hàng.
P/s: Mô hình khá đẹp, áp dụng phương tích sẽ ra. 

 

 




#727921 [TOPIC] ÔN TẬP HÌNH HỌC THI VÀO THPT CHUYÊN 2020-2021

Đã gửi bởi PDF on 07-06-2021 - 16:34 trong Hình học

$\boxed{45}$ (Đề thi thử Archimedes ngày 5/6/2021) Cho tam giác $ABC$ có hai đường cao $BE,CF$ cắt nhau tại $H$ ($E \in CA, F\in AB$). Gọi $M,N$ là trung điểm $CE,BF.$ $NM$ cắt $BE,CF$ tại $P,Q.$

a) Chứng minh rằng $\dfrac{PB}{PE}=\dfrac{QF}{QC}.$

b) Chứng minh rằng $(AMN)$ tiếp xúc $(HPQ).$

c) Gọi $I$ là trung điểm $PQ;$ K là giao điểm của $IH$ và $EF.$ Chứng minh rằng $(KPQ)$ đi qua trực tâm của tam giác $HPQ.$

Spoiler câu a, b

Gợi ý cho câu c: Chứng minh tứ giác $HPSQ$ điều hòa là xong.




#732991 BẤT ĐẲNG THỨC HƯỚNG TỚI KÌ THI CHUYÊN TOÁN 2021-2022

Đã gửi bởi PDF on 19-03-2022 - 22:03 trong Bất đẳng thức và cực trị

Một bài vui vẻ mà tôi bịa ra, đóng góp cho topic:

$\textbf{Bài 38.}$ Cho $a,b,c$ thực, không đồng thời bằng $0$.

Tìm giá trị nhỏ nhất và giá trị lớn nhất của

$$P=\frac{bc}{a^{2}+2b^{2}+2c^{2}}+\frac{ca}{b^{2}+2c^{2}+2a^{2}}+\frac{ab}{c^{2}+2a^{2}+2b^{2}}.$$




#732913 BẤT ĐẲNG THỨC HƯỚNG TỚI KÌ THI CHUYÊN TOÁN 2021-2022

Đã gửi bởi PDF on 13-03-2022 - 10:46 trong Bất đẳng thức và cực trị

Bài 26: Cho $a,b,c$ dương. Chứng minh rằng: $\frac{a}{b+c}+\frac{b}{c+a}+\frac{c}{a+b}\geqslant \frac{3}{2}+\frac{(a-b)^2}{2(a+b)^2}$

Bài này trong Group Hướng tới kỳ thi HSG và kỳ thi Chuyên Toán, có lẽ các tay to nhìn vào thì sẽ nói " Ối zời, SOS cứ thế mà phang thôi ", nhưng các bạn cứ SOS thử có khả quan không nhé

~~~~~~~~~~~~~~~

Lời giải.

Áp dụng bất đẳng thức Cauchy-Schwarz dạng phân thức: $\frac{a}{b+c}+\frac{b}{c+a}\geqslant \frac{(a+b)^2}{2ab+bc+ca}$

Ta cần chứng minh: $\frac{(a+b)^2}{2ab+bc+ca}+\frac{c(a+b)}{(a+b)^2}\geqslant \frac{3}{2}+\frac{(a+b)^2-4ab}{2(a+b)^2}\Leftrightarrow \frac{(a+b)^2}{2ab+bc+ca}+\frac{2ab+bc+ca}{(a+b)^2}\geqslant 2$

Bất đẳng thức cuối đúng theo AM - GM

Đẳng thức xảy ra khi $a=b=c$

$$(a+b)^{2}\left[\frac{a}{b+c}+\frac{b}{c+a}+\frac{c}{a+b}-\frac{3}{2}-\frac{(a-b)^{2}}{2(a+b)^{2}}\right]=\frac{a(a-c)^{2}}{b+c}+\frac{b(b-c)^{2}}{a+c}\geq 0,$$

$$\frac{a}{b+c}+\frac{b}{c+a}+\frac{c}{a+b}-\frac{3}{2}-\frac{(a-b)^{2}}{2(a+b)^{2}}=\frac{(a+b-c)^{2}(a-b)^{2}+(a^{2}+b^{2}-ac-bc)^{2}+2ac(a-c)^{2}+2bc(b-c)^{2}}{2(a+b)^{2}(a+c)(b+c)}\geq 0.$$




#732917 BẤT ĐẲNG THỨC HƯỚNG TỚI KÌ THI CHUYÊN TOÁN 2021-2022

Đã gửi bởi PDF on 13-03-2022 - 11:19 trong Bất đẳng thức và cực trị

Em không biết đây là kĩ thuật gì, anh có thể nêu cách tìm ra các hệ số trên không ạ?

Dự đoán dấu bằng xảy ra tại $a=b=c$, $VT-VP$ có bậc $3$ theo $c$, hệ số cao nhất là $a+b$ nên nhóm bình phương phần bậc $3$ thành $ac(a-c)^{2}+bc(b-c)^{2}$.

Phần còn lại là BĐT bậc $2$ theo $c$, có nhiều cách để phân tích.

Nhóm bình phương bậc $3$ như trên đòi hỏi kinh nghiệm nhất định.




#732262 [TOPIC] HÌNH HỌC

Đã gửi bởi PDF on 30-12-2021 - 21:34 trong Hình học

Bài giải đẹp, chất lượng nhưng có thể xử lí ngắn hơn bằng Brocard! 2 bài kia hình thức ngắn nhưng không biết khó không, để tui thử xem!

Bài 4: 

Cho tam giác $ABC$ nhọn có $AB<AC$. $E$ là điểm Nagel. Đường tròn $(I)$ nội tiếp tam giác tiếp xúc với $BC,CA,AB$ tại $M,N,P$. Qua $A$ kẻ đường thẳng song song với $BC$ cắt $MN,MP$ tại $R,S$. Đường tròn $(MRS)$ cắt $(I)$ tại $L$. Chứng minh $A,E,L$ thẳng hàng.

Gợi ý:




#732265 [TOPIC] HÌNH HỌC

Đã gửi bởi PDF on 30-12-2021 - 22:14 trong Hình học

Bài tiếp theo

Bài 5: Cho tam giác $ABC$ ngoại tiếp đường tròn $(I)$, nội tiếp đường tròn $(O)$. Điểm Nagel là $R$. Kẻ $AX//BC$ với $X$ thuộc đường tròn $(O)$. $K$ là giao điểm của đường tròn đường kính $AI$ với $(O)$. Chứng minh: $\angle KAI=\angle AXR$

Có thể sử dụng phép vị tự quay tâm $K$ và một bổ đề quen thuộc: Đường nối $A$ với tiếp điểm của đường tròn $A-$mixtilinear với $(O)$ đẳng giác với $AR$ trong góc $BAC$.




#732274 [TOPIC] HÌNH HỌC

Đã gửi bởi PDF on 31-12-2021 - 19:30 trong Hình học

Dạ anh nói đúng nhưng bài này khá nổi tiếng trong sách BÀI TẬP NÂNG CAO VÀ MỘT SỐ CHUYÊN ĐỀ HÌNH HỌC 10 của thầy Nguyễn Minh Hà, nếu anh có cách THCS thì có thể chia sẻ được không ạ?

Cách 1. Cách này anh nghĩ ra, không biết có đâu chưa

Còn 2 cách nữa, anh sẽ đăng sau.




#732275 [TOPIC] HÌNH HỌC

Đã gửi bởi PDF on 31-12-2021 - 19:36 trong Hình học

Bài 2: Cho tam giác ABC nhọn, các điểm $D,E,F$ lần lượt nằm trên các cạnh $BC,CA,AB$. Tìm vị trí của các điểm $D,E,F$ để chu vi tam giác $DEF$ nhỏ nhất.

Mở rộng cho tứ giác: Cho tứ giác $ABCD$ nội tiếp $(O)$ sao cho tất cả các góc tạo bởi một đường chéo và một cạnh có chung đỉnh của tứ giác đều là các góc nhọn. Tìm $4$ điểm trên từng cạnh của tứ giác sao cho chu vi tứ giác nội tiếp bên trong tứ giác $ABCD$ đạt giá trị nhỏ nhất. Chứng minh rằng có vô số tứ giác thỏa mãn điều kiện nhỏ nhất đó.




#732282 [TOPIC] HÌNH HỌC

Đã gửi bởi PDF on 31-12-2021 - 22:21 trong Hình học

Bài toán Fagnano - 1775:

Cách 2: Gọi $M,N$ lần lượt đối xứng với $D$ qua $AB,AC$. Khi đó $AM=AN=AD$ và $\angle MAN=2\angle BAC$.

Ta có $DE+EF+FD=EN+EF+FM\geq MN=2AM.\sin BAC=2AD.\sin BAC\geq 2h_a.\sin BAC$ với $h_a$ là đường cao xuất phát từ đỉnh $A$ của tam giác $ABC$.

Dấu bằng xảy ra khi $D,E,F$ là chân các đường cao của tam giác.

Đây chính là cách 2 anh muốn nói, của Fejer.

Còn 1 cách nữa của H.Schwarz.

Ngoài các cách trên ra, vẫn có thể còn các cách chỉ sử dụng kiến thức THCS đang chờ đợi. Hy vọng sẽ sớm được tìm ra.




#732272 [TOPIC] HÌNH HỌC

Đã gửi bởi PDF on 31-12-2021 - 19:02 trong Hình học

Bài này khá thâm, không phù hợp với THCS nên mình sẽ không giải vì nó cần phải sử dụng Vector

Em không nên đánh giá bài toán quá nhanh như vậy.

Anh nghĩ là sẽ có những người làm được bài này bằng cách THCS. Có ít nhất ba cách THCS.




#732311 [TOPIC] HÌNH HỌC

Đã gửi bởi PDF on 02-01-2022 - 17:31 trong Hình học

Bài đầu tiên trong năm 2022  :icon6:

Bài 8: Cho tam giác $ABC$ nhọn nội tiếp đường tròn $(O)$ có trực tâm $H$. $(BOC)$ cắt đường tròn đường kính $AO$ tại $P$, $(BHC)$ cắt đường tròn đường kính $AH$ ở $Q$. $AP$ cắt $(BOC)$ ở $I$, $AQ$ cắt $(BHC)$ ở $J$. Chứng minh $PQ//IJ$

Mở rộng: Với điều kiện mà Hoang72 đã nhắc tới trong lời giải của mình, $P,Q$ là hai điểm liên hợp đẳng giác trong tam giác $ABC$. Chú ý rằng từ sự kiện này ta có thể suy ra bài toán 8 mở rộng của Hoang72.




#730973 [TOPIC] BẤT ĐẲNG THỨC

Đã gửi bởi PDF on 05-10-2021 - 15:16 trong Bất đẳng thức và cực trị

Cho $a,b,c$ là $3$ số thực và $abc=1$. Chứng minh rằng:$\frac{1}{a^2+a+1}+\frac{1}{b^2+b+1}+\frac{1}{c^2+c+1}\geq 1$

 

                                                                                                 Các tiền bối giúp em bài này với ạ - làm cách dễ hiểu thôi ạ, em mới lớp 10 :))

Câu này trên mạng đầy lời giải.

Cho $a,b,c$ là các số thực dương. Chứng minh rằng:

$(a^2+ab+bc)(b^2+bc+ca)(c^2+ca+ab)\geq (ab+bc+ca)^3$

Cách 1. Không mất tính tổng quát, giả sử $b$ nằm giữa $a,c$.

Khi đó $$(b^{2}+bc+ca)(c^{2}+ca+ab)-(bc+ca+ab)(b^{2}+c^{2}+ca)=bc(a-b)(b-c)\geq 0,$$

$$a^{2}+ab+bc-(a^{2}+b^{2}+ac)=(a-b)(b-c)\geq 0.$$

Suy ra $$VT\geq (bc+ca+ab)(b^{2}+c^{2}+ca)(a^{2}+b^{2}+ca)\geq VP.$$

Cách 2.




#725623 [TOPIC] BẤT ĐẲNG THỨC

Đã gửi bởi PDF on 19-04-2021 - 07:47 trong Bất đẳng thức và cực trị

Không biết ý anh có phải như thế này không:

Áp dụng bất đẳng thức Cauchy, ta được: $a^3+b^3+c^3+9abc+4(a+b+c)\geqslant 2\sqrt{(a^3+b^3+c^3+9abc).4(a+b+c)}=4\sqrt{(a^3+b^3+c^3+9abc)(a+b+c)}$

Đến đây, ta cần chứng minh: $(a^3+b^3+c^3+9abc)(a+b+c)\geqslant 4(ab+bc+ca)^2$

Đặt $a+b+c=p;ab+bc+ca=q;abc=r$ thì ta có: $(a^3+b^3+c^3+9abc)(a+b+c)=(a^4+b^4+c^4)+[ab(a^2+b^2)+bc(b^2+c^2)+ca(c^2+a^2)]+9abc(a+b+c)=(p^4-4p^2q+2q^2+4pr)+(p^2q-2q^2-pr)+9pr=p^4-3p^2q+12pr$

Theo bất đẳng thức Schur bậc 3, ta có: $p^3+9r\geqslant 4pq\Rightarrow p^4+9pr\geqslant 4p^2q\Leftrightarrow p^4-3p^2q+12pr\geqslant p^2q+3pr$

Như vậy, ta quy về chứng minh: $p^2q+3pr\geqslant 4q^2$

Thật vậy, ta có: $p^2q+3pr=(a+b+c)^2(ab+bc+ca)+3abc(a+b+c)=(a^3b+ab^3)+(b^3c+bc^3)+(c^3a+ca^3)+2(a^2b^2+b^2c^2+c^2a^2)+8abc(a+b+c)\geqslant2(a^2b^2+b^2c^2+c^2a^2)+2(a^2b^2+b^2c^2+c^2a^2)+8abc(a+b+c)=4(ab+bc+ca)^2(Q.E.D)$ 

Có thể xử lý gọn hơn nữa như thế này:

BĐT tương đương với $$a^{4}+b^{4}+c^{4}+abc(a+b+c)+bc(b^{2}+c^{2})+ca(c^{2}+a^{2})+ab(a^{2}+b^{2})\geq 4(b^{2}c^{2}+c^{2}a^{2}+a^{2}b^{2}).$$

Áp dụng BĐT Schur bậc ba:

$$abc\geq (b+c-a)(c+a-b)(a+b-c)$$

$$\Rightarrow abc(a+b+c)\geq (a+b+c)(b+c-a)(c+a-b)(a+b-c)=2(b^{2}c^{2}+c^{2}a^{2}+a^{2}b^{2})-a^{4}-b^{4}-c^{4}$$

$$\Leftrightarrow a^{4}+b^{4}+c^{4}+abc(a+b+c)\geq 2(b^{2}c^{2}+c^{2}a^{2}+a^{2}b^{2}).$$

Lại theo BĐT AM-GM: $$bc(b^{2}+c^{2})+ca(c^{2}+a^{2})+ab(a^{2}+b^{2})\geq 2(b^{2}c^{2}+c^{2}a^{2}+a^{2}b^{2}).$$

Cộng lại có ngay đpcm. $\square$




#725577 [TOPIC] BẤT ĐẲNG THỨC

Đã gửi bởi PDF on 18-04-2021 - 00:03 trong Bất đẳng thức và cực trị

$\boxed{4}$Kỹ thuật dồn biến (Một bài dồn biến theo mình là cực hay)

Bài 42: Cho $a,b,c$ không âm. Chứng minh rằng: $a^3+b^3+c^3+9abc+4(a+b+c)\geqslant 8(ab+bc+ca)$

Cách dồn biến:

Đặt $$f(a,b,c)=a^{3}+b^{3}+c^{3}+9abc+4(a+b+c)-8(bc+ca+ab).$$

Ta có: $$\left[f(a,b,c)-f(a,\frac{b+c}{2},\frac{b+c}{2})\right][f(a,b,c)-f(a,b+c,0)]=-\frac{bc(b-c)^{2}(9a-3b-3c-8)^{2}}{4}\leq 0.$$

Do đó một trong hai thừa số phải không âm, tức là ta cần kiểm tra BĐT trong trường hợp có hai biến bằng nhau hoặc có một biến bằng 0.

Trường hợp 1: Hai biến bằng nhau.

Ta có $$f(a,t,t)=a^{3}+(9t^{2}-16t+4)a+2t(2-t)^{2}.$$

Rõ ràng ta chỉ cần xét trường hợp $9t^{2}-16t+4<0$, tức là $$\frac{8-2\sqrt{7}}{9}\leq t\leq \frac{8+2\sqrt{7}}{9}<2.$$

Áp dụng BĐT AM-GM, ta chỉ cần chứng minh

$$27t^{2}(2-t)^{4}+(9t^{2}-16t+4)^{3}\geq 0,$$

hay $$4(189t^{4}-648t^{3}+648t^{2}-160t+16)(t-1)^{2}\geq 0.$$

Với $t\leq 1$, BĐT là hiển nhiên. Xét $t>1$.

Áp dụng BĐT AM-GM có:

$$VT\geq 108\sqrt{42}t^{3}-160t+16>108\sqrt{42}t^{2}-160t+16\geq (48\sqrt[4]{378}-160)t>0.$$

Trường hợp 2: Có một biến bằng 0.

Ta có $$f(a,t,0)=a^{3}-4(2t-1)a+t(t^{2}+4).$$

Ta cũng dùng BĐT AM-GM để khử $a$ rồi chứng minh BĐT một biến giống trường hợp trên.

Đẳng thức xảy ra khi $a=b=c=1$ hoặc $a=0,b=c=2$ và các hoán vị tương ứng. $\square$

PS: Có cách khác không sử dụng dồn biến rất gọn và đẹp. :)




#725711 [TOPIC] BẤT ĐẲNG THỨC

Đã gửi bởi PDF on 21-04-2021 - 09:50 trong Bất đẳng thức và cực trị

$\boxed{52}$Cho $x,y,z$ là các số thực không âm thỏa mãn $3\leqslant x+y+z\leqslant 6$. Chứng minh rằng: $\sqrt{1+x}+\sqrt{1+y}+\sqrt{1+z}\geqslant \sqrt{xy+yz+zx+15}$

Phản ví dụ: $x=3,y=z=1$.

Edited: Nhầm :)




#725851 [TOPIC] BẤT ĐẲNG THỨC

Đã gửi bởi PDF on 24-04-2021 - 10:32 trong Bất đẳng thức và cực trị

Dạo gần đây, mình lo tập trung phát triển Box Hình học THCS nên không có thời gian quan tâm đến bất đẳng thức nói chung cũng như TOPIC của mình nói chung. Bây giờ sẽ là bài tiếp theo, các bạn cùng thảo luận:

$\boxed{54}$ Cho các số thực $a,b,c$ không âm thỏa mãn $\frac{a}{b+c}\geqslant 2$. Chứng minh rằng: $(ab+bc+ca)[\frac{1}{(a-b)^2}+\frac{1}{(b-c)^2}+\frac{1}{(c-a)^2}]\geqslant \frac{49}{18}$ 

Ta có bất đẳng thức mạnh hơn như sau:

Với mọi số nguyên không âm phân biệt $a,b,c$, ta có

$$\frac{1}{(b-c)^{2}}+\frac{1}{(c-a)^{2}}+\frac{1}{(a-b)^{2}}\geq \frac{4}{bc+ca+ab}.$$

Không mất tính tổng quát, giả sử $c$ là số nhỏ nhất trong ba số.

Khi đó $$VT\geq \frac{1}{a^{2}}+\frac{1}{b^{2}}+\frac{1}{(a-b)^{2}}=\frac{2}{ab}+\frac{(a-b)^{2}}{a^{2}b^{2}}+\frac{1}{(a-b)^{2}}\geq \frac{4}{ab}\geq \frac{4}{bc+ca+ab}.$$

Đẳng thức xảy ra tại bộ $(1+\sqrt{5},-1+\sqrt{5},0)$ và các hoán vị tương ứng. $\square$




#730994 [TOPIC] BẤT ĐẲNG THỨC

Đã gửi bởi PDF on 06-10-2021 - 09:04 trong Bất đẳng thức và cực trị

 

Xin góp hai bài này.

a, Cho tam giác nhọn ABC, chứng minh ${\left( {\frac{{cosA}}{{\cos B}}} \right)^2} + {\left( {\frac{{cosB}}{{\cos C}}} \right)^2} + {\left( {\frac{{cosC}}{{\cos A}}} \right)^2} + 8cosA.cosB.cosC \ge 4$

b, $x \in \mathbb{R};{\text{ }}n \in {\mathbb{Z}^ + }$, chứng minh $\left| {\sum\limits_{k = 1}^n {\frac{{\sin kx}}{k}} } \right| \leqslant 2\sqrt \pi  $, và đánh giá chặt hơn.

Hai bài này tôi lấy trong quyển "Arbelos 1982-1983", mà theo tôi thấy hay.

 

Theo tôi, câu a có vẻ hợp lý, nhưng câu b có vẻ không phù hợp để đăng ở đây lắm.

Lời giải câu b: https://math.stackex...1n?noredirect=1

Câu a:

Tồn tại $x,y,z>0$ để $\cos{A}=\sqrt{\frac{yz}{(x+y)(x+z)}},\cos{B}=\sqrt{\frac{zx}{(y+z)(y+x)}},\cos{C}=\sqrt{\frac{xy}{(z+x)(z+y)}}$.

Bất đẳng thức trở thành

$$\sum_{cyc} \frac{y(y+z)}{x(x+z)}\geq 4\left(\sum_{cyc}\frac{yz}{(x+y)(x+z)}\right).$$

Trừ hai vế cho $3$ ta có

$$\frac{(x+y+z)^{2}}{(y+z)(z+x)(x+y)}\left(\sum_{cyc}\frac{(y-z)^{2}}{y}\right)\geq \frac{1}{(y+z)(z+x)(x+y)}\left(\sum_{cyc}x(y-z)^{2}\right).$$

Mà $\frac{(x+y+z)^{2}}{y}>x$ nên ta có ngay đpcm.




#727072 [TOPIC] BẤT ĐẲNG THỨC

Đã gửi bởi PDF on 15-05-2021 - 13:51 trong Bất đẳng thức và cực trị

Góp cho topic 1 bài :
Cho $a,b,c>0$  $1/a+1/b+1/c=a+b+c$
CMR : $\sum 1/(2a+b+c)^2\leq 3/16$
 

Lời giải khác:

Sử dụng BĐT AM-GM:

$$(2a+b+c)^{2}=\frac{(6a+3b+3c)^{2}}{9}\geq \frac{4(2a+2b+2c)(4a+b+c)}{9}=\frac{8(a+b+c)(4a+b+c)}{9}$$

$\Rightarrow VT\leq \frac{9}{8(a+b+c)}\left(\frac{1}{4a+b+c}+\frac{1}{4b+c+a}+\frac{1}{4c+a+b}\right)\leq \frac{9}{32(a+b+c)}\left(\frac{1}{3a}+\frac{1}{3b}+\frac{1}{3c}+\frac{3}{a+b+c}\right)=\frac{3}{32}+\frac{27}{32(a+b+c)^{2}}$

Mặt khác, dễ thấy $a+b+c\geq 3$ nên $VT\leq \frac{3}{16}$.

Đẳng thức xảy ra chỉ khi $a=b=c=1$. $\square$




#730997 [TOPIC] BẤT ĐẲNG THỨC

Đã gửi bởi PDF on 06-10-2021 - 10:05 trong Bất đẳng thức và cực trị

Lời giải của anh quá ngắn gọn và chuyên môn, em nghĩ học sinh cấp 3 khó mà hiểu được. Em có một cách khác, 48h nữa sẽ đăng.

Em thích lời giải dài à, anh cũng có. Em cứ đăng đi rồi anh sẽ đăng sau. Bài này lỏng nên nhiều cách.

Anh sẽ đăng nhiều cách, rồi em chọn cái em thích.